Difficult CR

This topic has expert replies
Legendary Member
Posts: 2330
Joined: Fri Jan 15, 2010 5:14 am
Thanked: 56 times
Followed by:26 members

Difficult CR

by mundasingh123 » Wed May 18, 2011 3:02 am
In a political system with only two major parties, the entrance of a third-party radical candidate into an election race has historically damaged the chances of one of the two major candidates. This notion played out in the presidential race of 2000. In this important election the third-party extremist candidate attracted some of the voters who otherwise would have voted for one of the two major candidates, but not voters who supported the other candidate. Because the third-party candidate affected the two major candidates unequally, for reasons neither of them has any control over, the practice of allowing a third-party presidential candidate was unfair and should not be allowed in future national elections.
If the factual information in the above passage is true, which of the following can be most reliably inferred?

A)If, before the emergence of a third party, voters were divided equally between the two major parties, neither of the major parties is likely to capture much more than one-half of the vote.
B)If the political platform of the third party is a compromise position between that of the two major parties, the third party will draw its voters equally from the two major parties.
C)A third-party candidate will not capture the votes of new voters who have never voted for candidates of either of the major parties.
D)The political stance of a third party will be more radical than that of either of the two major parties.
E)The founders of a third party are likely to be a coalition consisting of former leaders of the two major parties.
I am having 1 bad time with CRs today . I have the OA . I want your explanations if you are sure u have nailed it .
I Seek Explanations Not Answers

User avatar
Legendary Member
Posts: 1101
Joined: Fri Jan 28, 2011 7:26 am
Thanked: 47 times
Followed by:13 members
GMAT Score:640

by HSPA » Wed May 18, 2011 3:20 am
mundasingh123 wrote:In a political system with only two major parties, the entrance of a third-party radical candidate into an election race has historically damaged the chances of one of the two major candidates. This notion played out in the presidential race of 2000. In this important election the third-party extremist candidate attracted some of the voters who otherwise would have voted for one of the two major candidates, but not voters who supported the other candidate. Because the third-party candidate affected the two major candidates unequally, for reasons neither of them has any control over, the practice of allowing a third-party presidential candidate was unfair and should not be allowed in future national elections.
If the factual information in the above passage is true, which of the following can be most reliably inferred?

A)If, before the emergence of a third party, voters were divided equally between the two major parties, neither of the major parties is likely to capture much more than one-half of the vote.
B)If the political platform of the third party is a compromise position between that of the two major parties, the third party will draw its voters equally from the two major parties.
C)A third-party candidate will not capture the votes of new voters who have never voted for candidates of either of the major parties.
D)The political stance of a third party will be more radical than that of either of the two major parties. [Third party grabs a share of one of the two.. it is not majority]
E)The founders of a third party are likely to be a coalition consisting of former leaders of the two major parties. [Out of scope] .
First take: 640 (50M, 27V) - RC needs 300% improvement
Second take: coming soon..
Regards,
HSPA.

Legendary Member
Posts: 2330
Joined: Fri Jan 15, 2010 5:14 am
Thanked: 56 times
Followed by:26 members

by mundasingh123 » Wed May 18, 2011 3:57 am
Hmm
I Seek Explanations Not Answers

Newbie | Next Rank: 10 Posts
Posts: 9
Joined: Fri Apr 08, 2011 6:47 am
Thanked: 4 times
GMAT Score:680

by jerseyny » Wed May 18, 2011 6:17 am
I think it is C...

In my opinion, the main point is that the third party is taking away the votes which belongs to one particular party if there was no third party. For instance, let's say democrats and republicans. Suddenly there is this super liberal party and this 3rd party candidate will take away democrats votes. However, can we firmly say that the 3rd party will always take away the votes? If someone is really liberal and doesn't like both parties, he/she might not vote at all. Therefore, I think C is the correct answer as the overall passage is assuming that if a candidate from 3rd party enters the race, the candidate will take away some parties' votes. English is not my first language so I could be wrong though..


In a political system with only two major parties, the entrance of a third-party radical candidate into an election race has historically damaged the chances of one of the two major candidates. This notion played out in the presidential race of 2000. In this important election the third-party extremist candidate attracted some of the voters who otherwise would have voted for one of the two major candidates, but not voters who supported the other candidate. Because the third-party candidate affected the two major candidates unequally, for reasons neither of them has any control over, the practice of allowing a third-party presidential candidate was unfair and should not be allowed in future national elections.
If the factual information in the above passage is true, which of the following can be most reliably inferred?

A)If, before the emergence of a third party, voters were divided equally between the two major parties, neither of the major parties is likely to capture much more than one-half of the vote.
B)If the political platform of the third party is a compromise position between that of the two major parties, the third party will draw its voters equally from the two major parties.
C)A third-party candidate will not capture the votes of new voters who have never voted for candidates of either of the major parties.
D)The political stance of a third party will be more radical than that of either of the two major parties.
E)The founders of a third party are likely to be a coalition consisting of former leaders of the two major parties.

User avatar
Master | Next Rank: 500 Posts
Posts: 135
Joined: Tue Mar 22, 2011 12:01 pm
Thanked: 21 times
Followed by:3 members
GMAT Score:720

by sourabh33 » Wed May 18, 2011 6:40 am
Huh!! what's the source?

IMO B

(Got this by eliminating other choices. However it is still difficult to find a strong co relation with the stimulus for this choice. The only reason option B looks valid is that the passage talks about voters,not the supporter, supporting the third party. In addition, the stimulus says the third-party candidate affected the two major candidates unequally, for reasons neither of them has any control over ). With this, we may assume that the author is trying to say that if the third party's policy are in between of the other two majors parties, it may draw its voters equally from the two major parties)

Still, in my opinion the stimulus has a very very poor co relation with the answer choices. I would pray 1000 times for not seeing such questions on exam day. :-(
Last edited by sourabh33 on Wed May 18, 2011 10:52 am, edited 1 time in total.

User avatar
Senior | Next Rank: 100 Posts
Posts: 37
Joined: Tue Jan 18, 2011 2:50 am
Thanked: 2 times

by abidshariff » Wed May 18, 2011 9:35 am
hmmm..difficult..very diff indeed....but i managed to cut down to 2 choices..it must be between A and B...
@jerseyny..IMO it cant be C because take a look at the sentence "the third-party extremist candidate attracted some of the voters who otherwise would have voted for one of the two major candidates,"..'some of the voters' means the third party might have attracted some who voted for neither.
And between A and B...I ll be more inclined towards A than towards B because the passage told that the third party is dividing the votes unequally. So B weakens the conclusion, for the passage dint suggest what type the third party is. We can still assume that it is a compromise, and the conclusion falls apart.

Moreover, say we have a total of 10 votes. Before the entry of a 3rd party, A has 5 and B has 5(as said in the choice A). Assume that all the votes to A have been voted by the supporters of A, whereas those to B have been voted by the people who supported neither, but just came there to vote for either of the two parties. So if the third party enters, it ll take away all the votes of B(assuming the extreme case). So now A has its 5 votes. And the 3rd party has 5 compared with 0 of B. So even after assuming the extreme case, A has 5 votes. So it can't have more than one-half the votes. So A IMO...

Legendary Member
Posts: 2330
Joined: Fri Jan 15, 2010 5:14 am
Thanked: 56 times
Followed by:26 members

by mundasingh123 » Wed May 18, 2011 11:14 am
Cool now that enuf ppl have contributed to the discussion , OA is B but the reason is not exactly what sourabh33 stated
Source BTG Practice questions
You may well pray that u dont see such quests on exam day .
I didnt reall get the logic of the OE so i am waiting for an expert to chime in
Ya abid all that quant was just futile
I Seek Explanations Not Answers

User avatar
Master | Next Rank: 500 Posts
Posts: 135
Joined: Tue Mar 22, 2011 12:01 pm
Thanked: 21 times
Followed by:3 members
GMAT Score:720

by sourabh33 » Wed May 18, 2011 11:22 am
So Exactly what is the reason for the OA

Legendary Member
Posts: 2330
Joined: Fri Jan 15, 2010 5:14 am
Thanked: 56 times
Followed by:26 members

by mundasingh123 » Wed May 18, 2011 12:27 pm
sourabh33 wrote:So Exactly what is the reason for the OA
Want to wait for for an Expert Opinion First , Excuse me for the delay
I Seek Explanations Not Answers

User avatar
Legendary Member
Posts: 1101
Joined: Fri Jan 28, 2011 7:26 am
Thanked: 47 times
Followed by:13 members
GMAT Score:640

by HSPA » Wed May 18, 2011 6:24 pm
mundasingh123 wrote:Cool now that enuf ppl have contributed to the discussion , OA is B but the reason is not exactly what sourabh33 stated
Source BTG Practice questions
You may well pray that u dont see such quests on exam day .
I didnt reall get the logic of the OE so i am waiting for an expert to chime in
Ya abid all that quant was just futile
I tried to google this and found OA as A using this https://www.beatthegmat.com/two-major-ca ... 51065.html .. The question I found has A/B swaped
First take: 640 (50M, 27V) - RC needs 300% improvement
Second take: coming soon..
Regards,
HSPA.

User avatar
Legendary Member
Posts: 516
Joined: Mon Nov 02, 2009 6:42 am
Location: Mumbai
Thanked: 14 times
Followed by:1 members
GMAT Score:710

by ankurmit » Wed May 18, 2011 9:34 pm
..
--------
Ankur mittal

GMAT/MBA Expert

User avatar
GMAT Instructor
Posts: 3380
Joined: Mon Mar 03, 2008 1:20 am
Thanked: 2256 times
Followed by:1535 members
GMAT Score:800

by lunarpower » Wed May 18, 2011 10:08 pm
this is a very, very badly written question; you are best off ignoring it.

first of all, the format is weird; the question is of the "draw conclusion" type, but the passage is actually an argument (normally, "draw conclusion" problems are based on passages that are just sets of facts, not arguments).

that's not the most serious problem, though.

the serious problem is that the facts in the passage are only true for the election of 2000, but the supposedly correct answer makes a prediction about future elections.

here's the passage:
mundasingh123 wrote:In a political system with only two major parties, the entrance of a third-party radical candidate into an election race has historically damaged the chances of one of the two major candidates. This notion played out in the presidential race of 2000. In this important election the third-party extremist candidate attracted some of the voters who otherwise would have voted for one of the two major candidates, but not voters who supported the other candidate. Because [xxxxx conclusion -- not a fact so irrelevant to drawing conclusions].
note that all of the factual information here deals with the past; there is absolutely nothing indicating that these patterns will continue into the future.
A)If, before the emergence of a third party, voters were divided equally between the two major parties, neither of the major parties is likely to capture much more than one-half of the vote.
notice the tense here -- "is likely to" clearly indicates that this choice is making a claim about present (or future) elections. if this conclusion were limited to the results of the 2000 election -- which it *must* be, in order to have any kind of validity in this sort of question -- then it would have to be expressed in the past (would have been likely, etc.)

this is especially troubling because one VERY, VERY common type of WRONG answer to conclusion questions is the type of answer that assumes that past or current behavior will continue into the future.
so, unfortunately, this problem is not only questionable at best, but actually training you to think in a way that produces some of the most common *incorrect* answers on the whole test.
ignore.
burn.
wash your hands.
Ron has been teaching various standardized tests for 20 years.

--

Pueden hacerle preguntas a Ron en castellano
Potete chiedere domande a Ron in italiano
On peut poser des questions à Ron en français
Voit esittää kysymyksiä Ron:lle myös suomeksi

--

Quand on se sent bien dans un vêtement, tout peut arriver. Un bon vêtement, c'est un passeport pour le bonheur.

Yves Saint-Laurent

--

Learn more about ron

Legendary Member
Posts: 2330
Joined: Fri Jan 15, 2010 5:14 am
Thanked: 56 times
Followed by:26 members

by mundasingh123 » Wed May 18, 2011 11:23 pm
---------------
I Seek Explanations Not Answers

Legendary Member
Posts: 2330
Joined: Fri Jan 15, 2010 5:14 am
Thanked: 56 times
Followed by:26 members

by mundasingh123 » Wed May 18, 2011 11:24 pm
OE
(B) is the credited response.

The suggestion offered in the paragraph is that a thirdparty candidate usually offers extreme viewpoints and a unique platform. By standing, in a political sense, to the far side of one of the major candidates, the third party candidate will attract a fringe percentage from that candidate. But the extremist won't draw votes from the rival candidate. Those voters discount both of the other candidates.
(B) offers an alternative notion that a third-party candidate who politically stands between the major party candidates will draw equally from both. This is a valid and sensible inference to draw. In the passage the third-party candidate is mentioned as a "radical" and an "extremist," which positions her or him to the far right or left. This is the tie-in between the passage and (B).

(A) is incorrect.

(A), a decent choice, speculates on the statistical outcome when a third-party candidate enters an election. While it might be true, if the third-party candidate has any following at all, that it will draw some votes, this choice is not as strong as (B).

(C) is incorrect.

(C) opens a new issue for discussion-uncommitted voters. While an enticing option, (C) introduces information not mentioned in the passage.

(D) is incorrect.

(D) is given in the paragraph as a fundamental characteristic of third-party candidates. It is not an inference.
I Seek Explanations Not Answers

GMAT/MBA Expert

User avatar
GMAT Instructor
Posts: 3380
Joined: Mon Mar 03, 2008 1:20 am
Thanked: 2256 times
Followed by:1535 members
GMAT Score:800

by lunarpower » Thu May 19, 2011 12:19 am
mundasingh123 wrote:The suggestion offered in the paragraph is that a thirdparty candidate usually offers extreme viewpoints and a unique platform. By standing, in a political sense, to the far side of one of the major candidates, the third party candidate will attract a fringe percentage from that candidate. But the extremist won't draw votes from the rival candidate. Those voters discount both of the other candidates.
(B) offers an alternative notion that a third-party candidate who politically stands between the major party candidates will draw equally from both. This is a valid and sensible inference to draw. In the passage the third-party candidate is mentioned as a "radical" and an "extremist," which positions her or him to the far right or left. This is the tie-in between the passage and (B).
um ... wow. is this actually the official explanation offered for the answer?

if so, then this is so bad that the credibility of the entire source is damaged.
if that is the official explanation, then this isn't just a case of a badly written problem -- it's a case in which the author of the problem fundamentally doesn't understand the way in which the test qusetions work. and, presumably, this question has managed to sneak past at least one editor, who therefore doesn't really understand how the test works either.

this answer choice is 100% pure speculation about a situation that is not mentioned in the passage at all, and about which NO logical deduction may be drawn from the passage. i.e., the passage contains no more information about this sort of situation than it contains about, say, what i ate for lunch yesterday.
in fact, the answer explanation contains the exact words that immediately DISqualify it: it "offers an alternative notion", i.e., a notion that is completely unaddressed in the scope of the existing passage.
so, this so-called "correct" answer is actually an answer that could be eliminated immediately on the test, for TWO different reasons
--> 1) this is an inference/conclusion question and it is making an inference that is completely outside the scope of the passage;
--> 2) it is using only past outcomes, but making a prediction about present or future events (i.e., assuming without justification that any relevant patterns will continue).

again, the fact that an editor let this problem slip by is very, very serious; unfortunately, from this point onward, you are going to have to look suspiciously upon any other questions from this source.

--

also, note that the explanation contains the phrase "credited response", a phrase generally used only on the LSAT (law-school exam) -- suggesting that the author is not overly familiar with the GMAT.

this observation doesn't excuse the horrible writing of the question -- this reasoning would also be incorrect if the question appeared on the LSAT -- but it puts the problem into more perspective, because the LSAT contains question types (that the GMAT does *not* contain) on which this problem could be correct.
for instance, the LSAT has contained questions that ask "which of the following is *consistent with* the statements in the passage?" -- a much weaker type in which the correct answer is simply the one that doesn't contradict the passage (rather than an answer that can actually be inferred from the passage).
*if* this were such a question, then this answer choice would be correct -- but it's not that type of question.

conclusion:
ignore this question.
from now on, be suspicious of this source.
Ron has been teaching various standardized tests for 20 years.

--

Pueden hacerle preguntas a Ron en castellano
Potete chiedere domande a Ron in italiano
On peut poser des questions à Ron en français
Voit esittää kysymyksiä Ron:lle myös suomeksi

--

Quand on se sent bien dans un vêtement, tout peut arriver. Un bon vêtement, c'est un passeport pour le bonheur.

Yves Saint-Laurent

--

Learn more about ron